0% found this document useful (0 votes)
43 views6 pages

(Ex) 6 (1) .1

1) The document defines lim sup and proves some properties about it. 2) It proves that lim sup an ≤ l if and only if an < l + ε for all n greater than some N, for any ε > 0. 3) It also proves that lim sup an ≥ l if and only if there exists a subsequence ank such that ank > l - ε for large nk, for any ε > 0.

Uploaded by

이퓨처
Copyright
© © All Rights Reserved
We take content rights seriously. If you suspect this is your content, claim it here.
Available Formats
Download as PDF, TXT or read online on Scribd
0% found this document useful (0 votes)
43 views6 pages

(Ex) 6 (1) .1

1) The document defines lim sup and proves some properties about it. 2) It proves that lim sup an ≤ l if and only if an < l + ε for all n greater than some N, for any ε > 0. 3) It also proves that lim sup an ≥ l if and only if there exists a subsequence ank such that ank > l - ε for large nk, for any ε > 0.

Uploaded by

이퓨처
Copyright
© © All Rights Reserved
We take content rights seriously. If you suspect this is your content, claim it here.
Available Formats
Download as PDF, TXT or read online on Scribd
You are on page 1/ 6

Lemma 1 Let {an } be a real sequence such that {an } is bounded above.

(a) lim sup an ≤ l if and only if for any (small) ² > 0, there exists an N = N (²) > 0 such that
n→∞
an < l + ² whenever n ≥ N . Here, l ∈ R.

(b) lim sup an ≥ l if and only if for any (small) ² > 0, there exists a subsequence {ank } of
n→∞
{an } such that ank > l − ² for nk À 1. Here, l ∈ R.

Denote by A the set of all subsequential limits of {an }. Since {an } is bounded above, it
follows that an ≤ M for some constant M > 0.

Proof of (a).
(⇒) We argue by contradiction and suppose that there exist a constant ²0 > 0 satisfying the
following property:

For any N > 0, there exists an n ≥ N such that an ≥ l + ²0 .

We claim that there exists a subsequence {ank } of {an } such that ank ≥ l + ²0 . Indeed,
there exists an n1 ≥ 1 such that an1 ≥ l + ²0 . Moreover, there exists an n2 ≥ n1 + 1 such that
an2 ≥ l + ²0 . Repeating this argument, we can choose an nk ≥ nk−1 + 1 such that ank ≥ l + ²0 .
Our claim is proved.
We claim that A contains an element x0 ∈ [l + ²0 , M ]. Indeed, {ank } ⊂ [l + ²0 , M ] and the
interval [l+²0 , M ] is compact. Then {ank } has a subsequence {bn } which converges to an element
in [l + ²0 , M ]. Let x0 = lim bn . Since l + ²0 ≤ ank ≤ M , it follows that l + ²0 ≤ x0 ≤ M .
n→∞
Notice that {bn } is a subsequence of {an }, and consequently, x0 is a subsequential limit of {an }.
Namely x0 ∈ A ∩ [l + ²0 , M ]. Our claim is proved.
Therefore, we conclude that l + ²0 ≤ x0 ≤ sup A = lim sup an ≤ l, which leads to a contradic-
n→∞
tion. ////
(⇐) We claim that sup A ≤ l + ² for any ² > 0.
Indeed, let ² > 0 be given. Choose any element x ∈ A. Since x is a subsequential limit of
{an }, there exists a subsequence {ank } of {an } such that lim ank = x. Then there exists an
nk →∞
N such that l − ² < ank < l + ² for nk ≥ N . Consequently x = lim ank ≤ l + ².
nk →∞
We have proved that x ≤ l + ² for any x ∈ A. Then it follows that sup A ≤ l + ². Our claim
is proved.
Since the choice of ² > 0 is arbitrary, we conclude that sup A ≤ l. ¤

Proof of (b).
(⇒) Note that sup A = lim sup an . Given ² > 0, there exists an element y ∈ A such that
n→∞
y > sup A − ² ≥ l − ². Since y is a subsequential limit of {an }, there exists a subsequence {ank }
such that lim ank = y. Since y > l − ², it follows that ank > l − ² for nk À 1. ////
nk →∞
(⇐) Let {ank } be a subsequence of {an } such that ank > l − ² for nk À 1. Since an ≤ M , it
follows that l − ² ≤ ank ≤ M for nk À 1. In other words, {ank } ⊂ [l − ², M ] for nk À 1. Since
[l − ², M ] is compact, {ank } has a subsequence {bn } such that {bn } converges to an element of
[l − ², M ].

1
Let x0 = lim bn . Note that x0 ∈ [l − ², M ] and x0 is a subsequential limit of {an }, in other
n→∞
words, x0 ∈ A. Therefore it follows that

lim sup an = sup A ≥ x0 ≥ l − ².


n→∞

Since ² > 0 is arbitrary, we conclude that lim sup an ≥ l. ¤


n→∞

Exercieses 6.1
(You may skip #2, #4 and all the problems related to lim inf except #15 and #16. Lemma 1,
n→∞
#7-(a), #9-(c), #14-(a) and #16 are strongly recommended. See Remark 3 as well.)

X
1. Let an = αn + iβn where αn , βn ∈ R. Show that |an | converges if and only if both
n=1

X ∞
X
|αn | and |βn | converge.
n=1 n=1
Solution. Note that |αn |, |βn | ≤ |an | ≤ |αn | + |βn | for all n. For the sake of convenience,
X n X ∞ Xn
we let sn = |αk |, tn = |βk | and un = |ak |. Then {sn }, {tn } and {un } are
k=1 k=1 k=1
monotonically increasing, and moreover, sn , tn ≤ un ≤ sn + tn for all n.
(⇒) Since {un } converges, {un } is bounded. Since sn , tn ≤ un , it follows that {sn } and
{tn } are both bounded above. Consequently both {sn } and {tn } are convergent.
(⇐) Since both {sn } and {tn } converge, they are bounded. Since un ≤ sn + tn , {un } is
bounded above. Consequently {un } converges. ¤

P∞
3. Suppose an > 0 for every n. Show that n=1 an diverges ⇔ for any integers M and N ,
PN +p
there exists an integer p such that n=N an > M .
n
X
Solution. Let sn = ak for simplicity. Since sn is monotonically increasing, {sn } diverges
k=1
if and only if sn → ∞.
(⇒) Let M, N ∈ N be given. Since sn → ∞, it follows that sn − sN → ∞. Consequently
N
X +p
there exists an integer p such that sN +p − sN > M . In other words, an > M .
n=N
(⇐) We argue by contradiction, and suppose that {sn } converges. Then {sn } is bounded
above, and there exists a constant M > 0 such that sn ≤ M for all n. Consequently, for
N
X +p
any integers N and p, an = sN +p −sN −1 ≤ sN +p ≤ M , which yields a contradiction. ¤
n=N

5. Let A be the set of subsequential limits of a complex sequence {an }. Show that A is closed.
Proof. We need to prove that A ⊂ A. Choose any x ∈ A. We need to find a subsequence
{ank } of {an } such that ank → x.
Let ² > 0 be given. Then there exists an element y ∈ A such that |y −x| < ²/2. Since y ∈ A,

2
we can choose a subsequence {amj } such that amj → y. Then there is an N = N (²) > 0
such that |amj − y| < ²/2 for mj ≥ N . Consequently

|amj − x| ≤ |amj − y| + |y − x| < ² for mj ≥ N.

We have proved that for any ² > 0 there is an N 0 = N 0 (²) such that |aN 0 − x| < ².
Therefore, for each k ∈ N, we can choose an nk ∈ N such that |ank − x| < 1/k. It is obvious
that ank → x as k → ∞, and {ank } is the desired subsequence. ¤

6. (left as an exercise: homework) Note that lim an = 0 if and only if lim |an | = 0.
n→∞ n→∞

7. (a) If an ≥ bn for every n, show that

lim sup an ≥ lim sup bn .


n→∞ n→∞

Proof. We consider the following three cases.


Case 1. lim sup an = l ∈ R: For any ² > 0, there exists an N = N (²) > 0 such that
n→∞
an ≤ l + ² for n ≥ N (see Lemma 1). Since bn ≤ an , it follows that bn ≤ l + ² for n ≥ N .
Then Lemma 1-(a) implies that lim sup bn ≤ l.
n→∞
Case 2. If lim sup an = ∞ then there is nothing to prove.
n→∞
Case 3. If lim sup an = −∞ then an → −∞. Since an ≥ bn it follows that bn → −∞, and
n→∞
consequently, lim sup bn = −∞. ¤
n→∞

∗ Remark 1. The conclusion also holds true if we assume that an ≥ bn for n À 1. ♣

∗ Remark 2. Recall that lim sup bn = l = lim inf bn if and only if lim bn = l.
n→∞ n→∞ n→∞

8. (a) Let {an } and {bn } be positive, bounded sequences. Show that
³ ´³ ´
lim sup(an bn ) ≤ lim sup an lim sup bn .
n→∞ n→∞ n→∞

Proof. Let A be the set of subsequential limits of {an bn }. Since {an } and {bn } are bounded,
it follows that {an bn } is bounded, and consequently A is also bounded. For simplicity, we
let l = lim sup an and m = lim sup bn . Clearly l, m ≥ 0.
n→∞ n→∞
Let ² > 0 be given. By Lemma 1, there exists an N = N (²) > 0 such that an < l + ²
and bn < m + ² for all n ≥ N . Then it follows that an bn < (l + ²)(m + ²) for n ≥ N .
Consequently, #7-(a) (Remark 1) implies that

lim sup(an bn ) ≤ lim sup[(l + ²)(m + ²)] = (l + ²)(m + ²).


n→∞ n→∞

Since ² > 0 is arbitrary, we conclude that lim sup(an bn ) ≤ lm. ¤


n→∞

3
9. (a) (left as an exercise: homework)
(c) (Modified problem) Suppose that {an } is real and lim an = a ≥ 0. For any real bounded
n→∞
sequence {bn }, show that
³ ´µ ¶
lim sup(an bn ) = lim an lim sup bn
n→∞ n→∞ n→∞

Solution. We consider two cases:


Case 1: a > 0
Let l = lim sup bn for simplicity. Let 0 < ² < a be given. Lemma 1 implies that
n→∞
(
there is an N1 = N1 (²) > 0 such that bn < l + ² for n ≥ N1 ,
there exists a subsequence {bnk } such that bnk > l − ² for nk À 1.

Moreover, there is an N2 = N2 (²) > 0 such that |an − a| < ² for n ≥ N2 . Let N =
max{N1 , N2 }. If n ≥ N then an > a − ² > 0, and consequently,
(
an bn < (a + ²)(l + ²) = al + ²(a + l + ²) for n ≥ N,
(1)
ank bnk > (a − ²)(l − ²) = al − ²(a + l − ²) for nk ≥ N.

Then it follows from Lemma 1 that

al ≤ lim sup(an bn ) ≤ al.


n→∞

³ ´µ ¶
Therefore, lim sup(an bn ) = al = lim an lim sup bn .
n→∞ n→∞ n→∞

Case 2: a = 0
Since {bn } is bounded, there exists a constant M > 0 such that |bn | ≤ M for all n. Then
it follows that 0 ≤ |an bn | = |an ||bn | ≤ M |an | → 0 as n → ∞. Consequently an bn → 0 as
n → ∞. Since −M ≤ lim sup bn ≤ M , we conclude that
n→∞

³ ´µ ¶
lim sup(an bn ) = lim (an bn ) = 0 = lim an lim sup bn .
n→∞ n→∞ n→∞ n→∞

12. Show that lim sup an = L, L finite, if and only if the following conditions hold: For any ² > 0,
n→∞
(i) There exists an N = N (²) > 0 such that an < L + ² for all n ≥ N ;
(ii) There exists a subsequence {ank } such that ank > L − ². ({nk } is infinite.)
Solution. #12 is a Corollary of Lemma 1. ¤

14. Let {an } be a complex sequence.


¯a ¯ X∞
¯ n+1 ¯
(a) If lim sup ¯ ¯ = L < 1 show that an converges absolutely.
n→∞ an n=1

4
Solution. Fix a constant
¯a ¯ r > 0 such that L < r < 1. Lemma 1 implies that there exists an
¯ n+1 ¯
N > 0 such that ¯ ¯ < r for n ≥ N (² = r − N ). In other words, |an+1 | < r|an | for
an
n ≥ N . Therefore, if n > N then

|an | < r|an−1 | < r2 |an−2 | < · · · < rn−N |aN |.



X
Since 0 < r < 1, it follows that rn−N |aN | converges. Then the comparison test shows
n=1

X ∞
X
that |an | also converges. Therefore an converges absolutely. ¤
n=1 n=1
¯a ¯ P
¯ n+1 ¯
Remark: If lim sup ¯ ¯ > 1 then we have no information on the convergence of an .
n→∞ an
For example, if we let a2n−1 = 1/2n and a2n = 1/3n then
|an+1 |
lim sup = ∞ > 1.
n→∞ |an |
See #16. However,

X 1 1 1 1 1 1
an = + + + 2 + 3 + 3 + ···
n=1
2 3 22 3 2 3

converges. ♣

15. Suppose |an | > 0 for every n. Show that


|an+1 | |an+1 |
lim inf ≤ lim inf |an |1/n ≤ lim sup |an |1/n ≤ lim sup .
n→∞ |an | n→∞ n→∞ n→∞ |an |

Proof. We prove the third inequality only. If |an+1 |/|an | is not bounded above, namely if
|an+1 |
lim sup = ∞ then there is nothing to prove.
n→∞ |an |
|an+1 |
Suppose that |an+1 |/|an | is bounded above, namely, lim sup = l for some l ∈ [0, ∞).
n→∞ |an |
Let ² > 0 be given. There is an N = N (²) > 0 such that |an+1 |/|an | < l + ² for n ≥ N .
Then, for any n > N ,

|an | < (l + ²)|an−1 | < (l + ²)2 |an−2 | < · · · < (l + ²)n−N |aN |, n ≥ N.

Consequently,
N
|an |1/n < (l + ²)1− n |aN |1/n
for any n ≥ N . Since (l + ²)1−N/n |aN |1/n converges to l + ² as n → ∞, #7-(a) (Remark 1)
implies that
N N
lim sup |an |1/n ≤ lim sup[(l + ²)1− n |aN |1/n ] = lim [(l + ²)1− n |aN |1/n ] = l + ².
n→∞ n→∞ n→∞

Since ² > 0 is arbitrary, we conclude that lim sup |an |1/n ≤ l. ¤


n→∞

5
∗ Remark 3. If lim |an+1 |/|an | exists, #15 implies that lim |an |1/n also exists. Moreover,
n→∞ n→∞

|an+1 |
lim |an |1/n = lim .
n→∞ n→∞ |an |

2n n!
For example, consider a sequence an = . Since
nn
an+1 2n+1 (n + 1)! nn 2nn 2
= n+1
· n = n
=¡ ¢n ,
an (n + 1) 2 n! (n + 1) 1 + n1

|an+1 | 2
it follows that lim = . Therefore lim sup |an |1/n = lim |an |1/n = 2/e.
n→∞ |an | e n→∞ n→∞

This fact is useful in finding a radius of convergence (Section 6.3). ♣

1 1
16. Define a sequence {ak } by a2k−1 = k
and a2k = k for every k. Show that
2 3
an+1 an+1
lim inf = 0, lim sup = ∞, lim inf (an )1/n = 0, lim sup(an )1/n = 0.
n→∞ an n→∞ an n→∞ n→∞

Solution. Note that




 a2k 2k
an+1  = k, n = 2k − 1,
= a2k−1 3
an 
 a 3k 1 ³ 3 ´k
 2k+1 = k+1 = , n = 2k.
a2k 2 2 2
Consequently an+1 /an → 0 if n = 2k − 1 → ∞, and an+1 /an → ∞ if n = 2k → ∞.
Therefore
an+1 an+1
lim inf = 0, lim sup = ∞.
n→∞ an n→∞ an

We also note that


 ³ ´k/(2k−1)
 (a2k−1 )1/(2k−1) = 1

, n = 2k − 1,
(an )1/n = ³ 1 ´1/22

 (a2k )1/(2k) = , n = 2k.
3
√ √
Consequently (an )1/n → 1/ 2 if n = 2k − 1 → ∞, and (an )1/n → 1/ 3 if n = 2k → ∞.
Therefore
1 1
lim inf (an )1/n = √ , lim sup(an )1/n = √ .
n→∞ 3 n→∞ 2
¤

You might also like